User Avatar
aaronchristopherhendrix708
Joined
Apr 2025
Subscription
Free
PrepTests ·
PT128.S2.Q21
User Avatar
aaronchristopherhendrix708
Saturday, Sep 30 2023

#help

Curious if anyone else agrees with this?

In blind review, I became suspicious of answer choice B when I read "too often", but I eliminated confidently when I read "excludes".

I read the first two as perhaps failing to emphasize community to the same degree (and the author does offer a value judgement on this), but excludes feels too strong. I don't see how any of these approaches could function without including at least some degree of the notion of community.

PrepTests ·
PT128.S2.Q20
User Avatar
aaronchristopherhendrix708
Saturday, Sep 30 2023

Ok... with all due respect to our fearless leader, J.Y., I think there is an error in his analysis here, which shows up in his discussion of both the correct answer (B), and the "trap" answer (E).

J.Y. says when discussing (A), "...in fact, it never differentiates...".

I think this is actually untrue. The stimulus tells us the method:

1. It predicts within a range of 2.5 points on the Richter Scale.

2.That range of 2.5 CAN be the difference between "marginally perceptible" and "considerable damage".

However, I don't think that means it can never differentiate, because we don't know where either of these thresholds lie on the scale.

For example, lets say an earthquake needs to register a 6.0 or above to cause "considerable damage", and a below a 4 is considered "marginally perceptible".

IF the method predicts an earthquake will register in the 3.5 - 6.0 range, then yes, in that case, the problematic ambiguity would be present.

However, if the method predicts an earthquake will register a 1.0 - 2.5, the method has predicted an earthquake with the same range as always, but the problematic ambiguity is not present.

Assuming it is an accurate prediction (not a point in dispute for the logic of this question) the method successfully differentiated between the two, because there is no possibility of the 1.0-2.5 earthquake causing "considerable damage".

Therefore, even if (E) used the exact same terms as the stimulus ("marginally perceptible" and "considerable damage"), it would still be incorrect, because the stimulus does not trigger the sufficient condition. The bar for "unlikely to be useful" is much higher here. It says it can never distinguish (which, again, in certain circumstances, the method in question can).

Answer choice (B) correctly sets the bar for unlikely to be useful at if it ever (even once) fails to differentiate.

So in my mind, there are redundant reasons to eliminate (E). The one J.Y. described is enough, but I believe it contains this larger logical flaw as well.

If someone thinks my analysis is incorrect, please let me know ASAP. I know I risk angering the LSAT Gods by disputing J.Y., and my only hope of survival is that I am correct.

#help

#feedback

PrepTests ·
PT147.S4.Q21
User Avatar
aaronchristopherhendrix708
Friday, Sep 29 2023

Anyone struggling to eliminate B, scroll to the very bottom to the comment by Sarah889 (someone else pointed this out).

It is really easy to get lost in this question, and forget that we are looking for a necessary assumption. On BR, I found myself stumbling around, looking for anything that could help me make sense of what was happening. Resetting focus on what is necessary reveals C pretty quickly.

PrepTests ·
PT142.S1.Q17
User Avatar
aaronchristopherhendrix708
Friday, Jul 28 2023

#help

I got this wrong timed and in BR because I was trying to support a direction of causality.

The correct answer does not support direction at all, it simply supports the possibility of causality at all. It could just as easily be used to support the opposite argument, that white glass was discovered by accident in the production of Han Purple.

Is this reasoning correct? And if so, is this common? It seems very odd for a causal reasoning question to essentially ignore the direction of causality in the correct answer choice.

PrepTests ·
PT149.S1.Q10
User Avatar
aaronchristopherhendrix708
Wednesday, Jul 26 2023

Another issue with C:

C is basically saying that there is no paradox to resolve, that it is a purely rhetorical construct.

Do RRE questions ever behave like this in other instances?

PrepTests ·
PT149.S1.Q10
User Avatar
aaronchristopherhendrix708
Wednesday, Jul 26 2023

This question is crazy. I got lucky and got it right timed, but missed (confidently!) on blind review.

I don't understand JY's issue with choice A, can someone explain?

He doesn't like the fact that A doesn't explain "how". But why do we need to? The prey species has survived this long BECAUSE there are so many of them. It doesn't need to specifically be an adaptive mechanism, but it can still be the reason they survived.

And it also doesn't require a huge leap of logic to answer "how". Maybe they are just an insanely fertile species that reproduces in such great numbers they could survive if they were bright pink.

The only weakness I see with A is that it is a completely unmodified comparative statement. My reasoning would be stronger if A had said "most species are FAR more populous".

However, even without, I still like A more than C. C means we have to interpret "seems unlikely" to mean "human understanding". To me this requires a far bigger leap than A.

PrepTests ·
PT149.S1.Q25
User Avatar
aaronchristopherhendrix708
Wednesday, Jul 26 2023

#help

JY says C "could be better because it lacks the second element in the sufficient condition.

I didn't actually even diagram it like this, I just bumped "popular" to the domain level, and was left with...

Ground Breaking ---> Critically Acclaimed

...which then matches perfectly with C, except that C simply lacks an additional descriptor at the domain level.

I do often find myself diagramming complex conditionals in a more "bare bones" fashion than JY does. It hasn't come back to bite me yet (that I have found) but I am curious if anyone else thinks that it is often reasonable to diagram a little bit more simply than JY does?

And to be clear, this is not a criticism of our great and fearless leader! His job is to teach principles and the theory behind them, so he probably always errs on the side of granularity. Those of us in the trenches can play these things a little more "fast and loose" once we have a solid command of the topic.

But again, if anyone disagrees, or has horror stories of when this type of thinking became a problem, please share!

PrepTests ·
PT149.S4.Q24
User Avatar
aaronchristopherhendrix708
Saturday, Aug 26 2023

#help

Can someone explain why it was "safe" to chain these two statements? I see how, after the fact, it works out. But technically, shouldn't "artist" been elevated to the level of domain?

Otherwise, I don't think we can really chain these. The negation of "unemployed artist" is not "employed artist". It's any person in the world who cannot be accurately described as an "unemployed artist".

Same with the second statement with "employed artist".

Again, I see how it works out. But this seems like the kind of thing that the LSAT would normally punish you for doing, especially this late in the section when you have to be on high-alert because they are so often baiting you into tiny mistakes like this, with a trap answer choice waiting.

I got to the correct answer without chaining them. Admittedly, it was a longer route. I initially eliminated all 5 choices. My second pass through, it "clicked". The first half of A ("if there are artists...") essentially does just elevate "artist" to the level of domain, so we can operate in a binary world of artists (either employed or not employed".

PrepTests ·
PT149.S1.Q23
User Avatar
aaronchristopherhendrix708
Wednesday, Jul 26 2023

#help

Curious if anyone thinks my interpretation of B is reasonable?

I selected B timed, but on BR I read it almost like one of those answer choices with a conditional statement.

"if you are regarded as a cruel tyrant, people living under you are more likely to view you unfavorably".

Like all such answer choices, the question is, "has the sufficient condition been met?".

In this case, no. In fact, the argument is trying to prove exactly the opposite, that the sufficient condition has not been met.

To be clear, I understand this is not a "technically correct" interpretation, and that there is no conditional statement in B. But this is one of those outlier questions where you kind of have to go into "survival mode" and just get there however you can.

If C strengthens, it's not by much. And I have seen very reasonable arguments from "LSAT Professionals" that C actually could just as reasonably be interpreted to weaken the argument.

Anyways... does anyone think that "pseudo-conditional" reading of B is a reasonable way to approach it? Or is just just completely off?

PrepTests ·
PT139.S1.Q12
User Avatar
aaronchristopherhendrix708
Wednesday, Apr 26 2023

#help

Curious if any other students approached this without diagramming all the formal logic? I missed this timed, but in Blind Review the answer really jumped out at me, and fairly quickly. Without diagramming, or even thinking about the formal logic aspect, we can clearly see the word "acknowledge" attached to both sincerely apologizing and accepting an apology.

Did anyone else do this? Or do you think this is a fundamentally dangerous approach to a question like this?

My default is still to use diagramming formal logic as a backup plan when my first pass through the question doesn't reveal anything, but that is something I may need to work on.

PrepTests ·
PT139.S1.Q5
User Avatar
aaronchristopherhendrix708
Tuesday, Apr 25 2023

#help

Would it be fair to say another reason choice A is bad is because of the term "decaffeinated beverages"?

When I originally did this question, I interpreted that as "non-caffeinated" beverages and used that to eliminate (why do I care how much orange juice someone in this study drinks?).

I realize now that is incorrect, as "decaffeinated" obviously means something that has had its caffeine removed. However, it still works the same way to eliminate choice A.

What if this question referred to "decaffeinated tea"? That would have no impact on the evaluation of the study/argument.

All the other choices refer to either coffee specifically, or caffeine directly.

PrepTests ·
PT127.S3.Q23
User Avatar
aaronchristopherhendrix708
Friday, Jun 23 2023

#help

I eliminated D on the basis of "not enough information". I read that as we just don't know the wind conditions of the afternoon, and thus there is no way this could conform to the principle of the stimulus.

Are we supposed to assume the "intermittent wind" of the late morning continued into the afternoon?

I realize for the purpose of getting this question right, this discussion is mostly irrelevant, especially when you get to E. I just want to make sure I have a very particular understanding of how we are supposed to interpret D. Assuming that wind continues into the afternoon feels like exactly the type of thing the LSAT usually punishes you for, particularly because, in this case, each of the other answer choices leaves no ambiguity as to the state of the wind in the afternoon.

PrepTests ·
PT152.S4.Q7
User Avatar
aaronchristopherhendrix708
Sunday, Oct 22 2023

#help

Curious if anyone else approached this like I did?

I didn't actually see an argument here. It is more just 3 distinct claims, only one of which brings any evidence to the table (the 3rd).

I saw this as 3 distinct opportunities to support the stimulus, and each incorrect (supporting) answer choice does address one of them individually. A, C, D, E address 3, 2, 1, 2 respectively.

So I thought of these individually as just "premise boosters", as there isn't a clear argument structure to support or undermine.

Also, it does seem the LSAT is careful about word choice in the question stem. Here we are asked to support the nutritionists VIEW, not argument.

PrepTests ·
PT119.S3.Q20
User Avatar
aaronchristopherhendrix708
Saturday, Apr 22 2023

#help

Would it be fair to say choice A is also a Sufficient Assumption?

PrepTests ·
PT126.S1.Q24
User Avatar
aaronchristopherhendrix708
Wednesday, Jun 21 2023

#help

Disclaimer: I am not one of the "JY Haters" that roam these comments. I am a big fan of the man. If I wasn't, I would not have spent countless hours of my life listening to him talk about the LSAT.

That being said... does anyone else feel like on this particular question he overcomplicated things?

To me, the error can be really succinctly. The argument takes an (admittedly weak) correlation and then claims it is a sufficient condition for weight loss.

All you have to do is demonstrate that the claim is not universally true (C does this). Answer choice D is pretty weak ("some people"), but because the conclusion is SO forceful, it is particularly vulnerable.

I left JY's video much more confused than I went in, but I am very open to the possibility that I am missing something important, so anyone thinks that is the case, please share!

PrepTests ·
PT146.S4.P4.Q23
User Avatar
aaronchristopherhendrix708
Friday, Oct 13 2023

#help

Can someone explain why the correct answer for 23 is supported? The only way I see to get this right is to recognize that the fact that A, B, C, and D are pretty easy to eliminate. If it had said "what element of certain CFC's is most damaging", I would have no issue.

But we don't know anything about CFC's in general, just two specific kinds.

This is part of a larger frustration I have with RC, there are questions like this that just feel sloppy. An LR question would never leave a hole like this.

Am I misunderstanding something?

PrepTests ·
PT151.S4.Q19
User Avatar
aaronchristopherhendrix708
Friday, Aug 11 2023

#help

The way I employed conditional logic for this question was much simpler than what JY did.

I am curious if anyone thinks I just got lucky? Or is this a simpler, viable path than what JY did?

One quick read of the stimulus reveals that this whole question never steps outside the "Government Practice", so that gets kicked up to the domain level.

After that, all I did was use the "unless translation" with the word except. That gave me

FA----->CR

(Facilitate Abuse of Power------>Must have compelling Reason)

The stimulus tells us that concealing the existence of a secret = facilitation of abuse of power.

Meaning that C triggers the sufficient condition, irrespective of the fact that the secret itself was justifiably kept.

So the difficult of this question, in my mind, was not the underlying logic. That seems to me to be pretty simple. The difficulty is the wording, and the need to make subtle distinctions between "facilitating abuse of power" and "abusing power" (trap answer choice D). Or "keeping a secret" and "keeping a secret a secret" (a red herring built into the correct answer choice).

I would love any thoughts on my process, particularly if I oversimplified something.

PrepTests ·
PT136.S2.Q13
User Avatar
aaronchristopherhendrix708
Tuesday, Jul 11 2023

#help

I see now why A is the most accurate description, but I would love some feedback on C.

I think I got distracted by how stupid Morgenstern's response is, and I wanted to attack his premises. However, he is making some huge assumptions, is he not?

He offers no support for a pretty wild claim that "the only risk is ". I wish I could articulate my question more cleanly, but basically, what I am asking is...

Isn't a response that is this wildly unsupported more or less a mischaracterization of what Brooks has said?

Brooks makes a very reasonable statement, and Morgenstern completely dismisses it. I know the specific logical flaw is described in A, but you have to burrow past the larger issues to even get there.

To be clear, my point is not to argue that C is the correct answer. I just think this question leaves some interpretive ambiguity this test usually avoids.

More practically, any thoughts on how to avoid falling into this in the future?

Looking back, I definitely saw the logical flaw described in A. I got distracted by the real-world absurdity of what Morgenstern said, and felt C more holistically addressed it.

PrepTests ·
PT124.S1.Q2
User Avatar
aaronchristopherhendrix708
Saturday, Jun 10 2023

#help

Can someone help me understand the nature of these question types a bit more?

On my first pass, B was an easy eliminate, because it "feels" like a classic no-opinion for Frederick.

To accept B as an answer choice, we have to assume that his EXPLICIT goal of devoting limited resources is incompatible with the goal of seeking to represent all genres. To me, there is nothing to suggest that these 2 goals are incompatible. Fredericks explicit goal could just be his top priority, his "tiebreaker", but certainly it doesn't seem unreasonable to think he could have multiple goals.

Basically, we are being asked to view the phrase "should seek to" as a singular thing. You can pick one, and only one. This feels like the type of assumption we are punished for making elsewhere on the LSAT.

Zooming out from the specifics of the question itself, is it common to have to make an inference like this in these PIA questions?

PrepTests ·
PT135.S2.Q15
User Avatar
aaronchristopherhendrix708
Friday, Jun 09 2023

#help

I ended up with the correct chain of premises. However, instead of taking the contrapositive, I just left it in its original form. Answer choice D (CBI--->/C) means that if countries put collective before the individual needs, the people won't have confidence, and thus a necessary condition for "Rapid Emergence" is failed (meaning rapid emergence isn't possible).

I am by no means arguing this is a superior method. In fact, if you think this is clunky or dangerous, please share! But I mainly want to know, is this logically CORRECT? Or did I just get lucky and stumble my way to the right answer?

PrepTests ·
PT155.S1.Q8
User Avatar
aaronchristopherhendrix708
Monday, Oct 09 2023

#help #feedback

It ultimately doesn't matter for the purposes of the question, because clearly you can get to the correct answer either way, but isn't there a mistake here the way JY sets up the initial conditional statement?

Just reading intuitively, the first sentence means that having BOTH a High School Diploma AND Competence in CPR (HSD and CCPR) will not be licensed as an EMT (LEMT).

LEMT→ HSD + CCPR

(contrapositive)

/HSD or /CCPR→ /LEMT

So isn't it incorrect to have /HSD and /CCPR in the sufficient condition instead of or there?

PrepTests ·
PT148.S1.Q5
User Avatar
aaronchristopherhendrix708
Sunday, Jan 07 2024

#help

I think there is actually a better reason to eliminate D.

For D to be necessary, A would have to be necessary.

D is a clever trap answer choice because it is playing on the assumption of A. If we make that assumption without realizing it, then D has at least a decent argument for being necessary (enough for us to take the bait under timed conditions, at least).

Another way of saying this is if A assumed, then D is irrelevant, because they can get the protection they need from elsewhere, and their fate is decoupled from the fate of the grass.

PrepTests ·
PT158.S3.Q24
User Avatar
aaronchristopherhendrix708
Friday, Jan 05 2024

#help

Is this kind of a strange question? As I interpret it, D is just a direct attack on a premise.

I feel like a typical question would leave the last phrase "it cannot have effects significant enough to be worrisome would be left unstated".

If the argument said "there are no risks because the change is minimal", then this would feel much more typical, because we would then have to fill in the assumption, which D calls out.

But because it explicitly states the assumption, it then feels like a premise, and answer choice D just attacks it directly.

PrepTests ·
PT144.S3.Q2
User Avatar
aaronchristopherhendrix708
Saturday, Aug 05 2023

#help

1. How would you describe this flaw? It seems like we are just attacking the premise of the argument.

Columnist says "tools can't be the problem".

We respond with "yes they can, by being bad tools".

I know there is a little more nuance than that, but not much.

2. Does anyone else feel like the first 5-6 questions of any LR sections are sometimes... for lack of a more eloquent description... just weird?

I feel like at least one time a test in I will get an LR question in the first 6 that absolutely throws me. Not because the logical structure is complex, or the stimulus is particularly dense/convoluted, but because the question just doesn't seem to play by the same set of rules as other questions in it's type.

This is a classic example of that for me. I knew the columnists argument was terrible immediately, but for the life of me I couldn't find a way to articulate a flaw. I only got to the right answer by eliminating the other ones.

PrepTests ·
PT121.S1.Q7
User Avatar
aaronchristopherhendrix708
Thursday, Jun 01 2023

#help

Ultimately it doesn't really matter, as they are similar enough to get to the right answer regardless, but can someone explain why the last sentence is the conclusion, and not the sub-conclusion?

It seems to support the opening sentence, but the opening sentence offers no support for the closing sentence.

Confirm action

Are you sure?